שינויים

שיחה:88-133 אינפי 2 תשעב סמסטר ב/אינטגרלים

נוספו 239 בתים, 19:26, 10 באפריל 2012
/* atan */ פסקה חדשה
וכו' - אין לי יכולת אפילו לגשת לבעיה. (אבל אינטואיטיבית האינטגרביליות והחסומות תהיינה כנראה שתיים בחזקת אלף)
 
== atan ==
 
<math>\int_{0}^{-1}\frac{1}{1+x^2}dx=arctan(-1)=\left\{\begin{matrix}
-\frac{\pi}{4} \\
\frac{3\pi}{4}
\end{matrix}\right.</math>
 
וולפראם אומר שהראשון. זה בגלל האי-רציפות באמצע? למה?
410
עריכות